1answer.
Ask question
Login Signup
Ask question
All categories
  • English
  • Mathematics
  • Social Studies
  • Business
  • History
  • Health
  • Geography
  • Biology
  • Physics
  • Chemistry
  • Computers and Technology
  • Arts
  • World Languages
  • Spanish
  • French
  • German
  • Advanced Placement (AP)
  • SAT
  • Medicine
  • Law
  • Engineering
8_murik_8 [283]
3 years ago
9

A 40-year-old woman presents to the emergency room with flank pain and fever of 102°f. upon interview of the patient, she also n

oted that her urine output has decreased over the last few days. on further inquiry, the patient stated that she has been trying to lose weight by increasing her protein in her diet, exercising and decreasing her normal fluid intake. on urinalysis, red blood cells are noted. what is the most likely diagnosis?
Health
1 answer:
MrRa [10]3 years ago
5 0
I believe that she has Oliguria. It is the medical term for a decreased output of urine. Aside from that, there are many potential causes and the two that we see her are dehydration and a urinary tract obstruction. She is decreasing her normal fluid intake. Your body can't replace the fluids that you're losing. When this happens your kidneys retain as much fluid as they can. As for urinary tract obstruction, she has increased her protein intake. A high-protein diet can inhibit your body from eliminating all the waste products of protein metabolism. This can cause a blockage in your kidneys. This explains why she has flank pain and a high fever. 
You might be interested in
Contractions which cause the muscle to change length as it contracts and causes movement of a body part are:
defon

help me help me help me help me help me help me help me help me help me help me with mines pl

3 0
3 years ago
Read 2 more answers
6. Over crowded room is......
Zepler [3.9K]

Answer: two opposite sex adolescents occupying one room​.

Explanation:

A overcrowded room is a room that has fewer bedrooms for the occupants in the room which is typically based on the sex and age of the occupants.

Therefore, an overcrowded room is two opposite sex adolescents occupying one room​. In this case, since they are opposite genders and are matured enough, they should not be staying in a room together but rather have their own separate rooms. Therefore, in this case, the room is overcrowded.

4 0
3 years ago
What term defines the greatest force that can be produced by a muscle or group of muscles?
Vinil7 [7]
Muscular strength/endurance would be the term.

Hope this helped.
3 0
3 years ago
what muscle supports posture, protects internal organs , and provides movement from the pelvis to the ribs?​
rewona [7]

Answer:

The abdominal muscles

Explanation: The abdominal muscles are located between the ribs and the pelvis on the front of the body. The abdominal muscles support the trunk, allow movement and hold organs in place by regulating internal abdominal pressure.

3 0
3 years ago
Read 2 more answers
A junior-level nursing class has just finished learning about the management of clients with chronic pulmonary diseases. They le
ra1l [238]

Answer:

Emphysema.

Explanation:

COPD (Chronic obstructive pulmonary disease) is the medical condition that causes inflammation in lungs and makes breathing problematic. Chronic bronchitis and Emphysema are the two main conditions included in COPD.

Emphysema is the extreme lung condition of COPD that causes the breath shortness. The alveoli are completely damaged and also reduces the amount of oxygen in the body. The emphysema condition causes the decrease in the lung surface area and reduces the amount of oxygen.

Thus, the answer is emphysema.

5 0
3 years ago
Other questions:
  • Which of these actions is an example of practicing delayed gratification rather than acting on an impulse
    13·2 answers
  • How can i control bad Vocal tics <br><br> aka (Tourette syndrome)
    12·1 answer
  • A 15-year-old girl realizes that the dress she has worn to school has a small stain on it. Her belief that everyone will notice
    14·1 answer
  • Staying ______ helps to keep the metabolic rate high.
    13·2 answers
  • Myasthenia gravis is a disease resulting from an autoimmune attack on the ach receptors of the motor end plate. Binding of antib
    11·1 answer
  • Maurice plans to become a professional wrestler and wants to build his strength. Which is the best strategy for Maurice?
    15·2 answers
  • Brandon tends to suppress or bottle in his feelings. He is likely to experience
    8·2 answers
  • Heyyyy who wants brainliest???????/???
    6·1 answer
  • A healthy community have good health resources in place and can often recover more quickly after a disaster. True or false
    7·1 answer
  • 1. Compare the ways that society ( friend and local customs) influences your food choice
    14·1 answer
Add answer
Login
Not registered? Fast signup
Signup
Login Signup
Ask question!